Question about LR Q on December 09 Test Forum

Prepare for the LSAT or discuss it with others in this forum.
Post Reply
ZombiesAhead

New
Posts: 72
Joined: Thu Sep 24, 2009 8:47 pm

Question about LR Q on December 09 Test

Post by ZombiesAhead » Tue Jan 05, 2010 6:34 am

Can anyone help me with question #19 in LR2 from December? It's a must-be-true about babies, dictionary definitions, and understanding words. AC E makes sense to me after the fact but can this be diagrammed somehow?

User avatar
Atlas LSAT Teacher

Bronze
Posts: 283
Joined: Tue May 12, 2009 10:18 am

Re: Question about LR Q on December 09 Test

Post by Atlas LSAT Teacher » Tue Jan 05, 2010 12:54 pm

This is a tricky question! I wouldn't go for diagramming this, as I think it falls apart more quickly staying within the content of the argument and applying the statements.

If I were to diagram, I might use this:

[U = understanding a word; KD = know a definition; KW = know all words in a def; B= babies!]

(U --> KD) --> (U --> KW)
B (all) --> ~ KW (for some words)

The most important thing here is to notice that the first statement is NOT stating that (U --> KD), it simply is playing out the hypothetical effect of that being true. But, we can infer something based on that, since that hypothetical effect is stated as a fact: If we turn it into this: (A --> B) --> (A --> C), what is missing? It requires (assumes) B --> C. So, we know that (KD --> KW).

Another issue is that if the hypothetical were true, we would be able to infer by combining the two statements. Since we know that B --> ~ KD (for some words), we would know that, for babies, ~ U (again, for some words -- not for some babies), or simply B --> ~ U (some). But, again, we don't know that is the case, because we don't know whether the if condition of the first statement is true.

Answer choice (E) states that (B (some) --> KW (all words)) --> ~ (U --> KD). This is because we already know that all babies do know some definitions, and so if there's a baby that understands all the words it utters, it must not require knowing the definitions. Going back to the diagram, (E) contradicts our hypothetical inference, because if U and ~ KW are both true, then the second part of the hypothetical , (U --> KW), can't be true, and therefore the sufficient part of that initial hypothetical statement (U --> KD) cannot be true, which means you should be able to have U without KD.

From a test-smarts point of view, (E) "triggers" the "all babies" rule, so in a pinch, this would be a smarter guess than some other answer choices.

More intuitively, working from wrong to right:

(A) is extremely tempting! But we don't know this; we only know that babies don't know the definitions of some words they utter. IF we knew that knowing the definition was necessary for understanding a word (if we assumed that the hypothetical conditional statement were true), this would be a valid inference.
(B) we haven't learned that the initial rule is not true, though that definitely is where the stimulus would seem to naturally lead. It's also suspicious wording "Any number of people"
(C) this might be true -- babies are uttering some words for which they don't know the definitions, however we don't know that they understand a thing they are saying.
(D) from sheer mental exhaustion this answer choice might be tempting! However, it's negating the sufficient side of the initial statement, which does not mean we can negate the necessary side. In common sense talk: just because knowing a definition requires knowing the words in the definition (KD --> KW), it doesn't mean that if you can understand a word without knowing it's dictionary definition ~(U --> KD) without knowing the meaning of any other words in a language!

I hope that helps. I want to cook up an analogy for this argument -- but I have to take a break!

Post Reply

Return to “LSAT Prep and Discussion Forum”